In two paragraph of lines each, describe how you
would simplify, for your
grade 10 learners the
following topics
1.1 Electricity and magnetism.

Answers

Answer 1

The question above wants to assess your ability to simplify complex matters. In that case, I can't answer this question for you, but I'll show you how to answer it.

First, you should research electricity and magnetism. This research will make you understand the subject and all its elements. You can do this research in textbooks and digital platforms aimed at children and teenagers since these media usually present subjects in a more simplified way.

After doing this research, you will be able to write a text about electricity and magnetism as follows:

Present the main focus of this subject.Present the elements that make up this subject.Present how this subject applies in the real world.Present the importance of this subject in human life and society.

Remember that you must use simple language, without the use of technical terms, but with common terms.

More information:

https://brainly.com/question/14870576?referrer=searchResults


Related Questions

Write the number below in scientific notation, 35,600 O A. 3.56 x 103 O B. 3.56 x 103 O C. 3.56 x 104 O D. 3.56 x 105​

Answers

C. 3.56 x 10^4. There are 4 places right from the decimal point. 3.5600

PLEASE HURRY!!
WILL GIVE BRAINLIEST!!!!!!!!
Which of the following types of stars had the smallest initial mass?

Blue main sequence
Neutron star
Red supergiant
White dwarf

Answers

Answer:

the answer is White dwarf

Explanation:

The drawing shows a person looking at a building on top of which an antenna is mounted. The horizontal distance between the person’s eyes and the building is 85.5 m. In part a the person is looking at the base of the antenna, and his line of sight makes an angle of 38.9o with the horizontal. In part b the person is looking at the top of the antenna, and his line of sight makes an angle of 41.6o with the horizontal. How tall is the antenna?

Answers

height of building (h1):
tan(38.9°) = opposite / adjacent
tan(38.9°) = h1 / 85.5
85.5 tan(38.9°) = 68.990 (5sf)

height of building with antenna (h2):
tan(41.6°) = opposite / adjacent
tan(41.6°) = h2 / 85.5
85.5 tan(41.6°) = 75.910 (5sf)

height of antenna (h2 - h1)
75.910 - 68.990 = 6.92m

What are some of the strategies that you currently use to make consumer buying decisions

Answers

Answer: hope you find the explanation useful!

Explanation:

Consumer behavior considers numerous reasons why individual, situational, mental, and social individuals look for items, purchase, use, and afterward discard them.

Evaluative criteria are positive qualities that are imperative to you, for example, the cost of the knapsack, the size, the number of compartments, and shading. A portion of these attributes are valued highly as compared to others. For instance, the size of the rucksack and the cost may be more imperative to you than the shading except if, state, the shading is hot pink, and you despise pink.

A motorcycle travels 90.0 km/h. How many seconds will it take the motorcycle to cover 2.10 x 103 m?

Answers

Answer:

The time taken is 84 seconds, please give the brainliest award would be much appreciated.

Explain what would happen if an ebonite rod charged with fur is brought together with an acetate rod charged with silk

Answers

Answer:

The ebonite becomes negatively charged, and the fur becomes positively charged.

Explanation:

When an ebonite rod is rubbed against animal fur, some of the electrons from atoms of the fur are transferred to the rod.The ebonite becomes negatively charged, and the fur becomes positively charged.(a) A positive charge (+) and a negative charge (–) attract each other. (b) Two negative charges repel each other. (c) Two positive charges repel each other.

please answer me!!! asappp​

Answers

Answer:

omg this is hard

Explanation:

Yes it is lol

Which statement about magnetic force is FALSE?

Answers

put options for the answers

What happens if hair has static electricity

Answers

Answer:

it goes up

Explanation:

Static hair occurs when your hair builds up an electric charge, meaning it has gained some extra electrons. That makes the strands of your hair repel each other, leaving your hair frizzy and difficult to style.

If an object is falling towards the ground with a force of 98 N and accelerates at a rate of 9.8 m/s², what is the mass of the object (mass will be in the unit of kilograms)?​

Answers

Answer:

960.4

Explanation:

98x9.8=960.4


A bucket crane consists of a uniform boom of mass
M = 201 kg and length L= 59.05 ft that pivots at a point on
the bed of a fixed truck. The truck supports an elevated bucket
with a worker inside at the other end of the boom, as shown in
the figure. The bucket and the worker together can be modeled
as a point mass of weight 201 lb located at the end point of
the boom.
Suppose that when the boom makes an angle of 68.7° with the
horizontal truck bed, the bucket crane suddenly loses power,
causing the bucket and boom to rotate freely toward the
ground. Find the magnitude of the angular acceleration la of
the system just after the crane loses power. Take the rotation
axis to be at the point where the boom pivots on the truck bed.
Use g = 9.81 m/s2 for the acceleration due to gravity. For unit
conversions, assume that 1 m = 3.28 ft and 1 lb = 4.45 N.
Express your answer to at least two decimal places.

Answers

Newton's second law for rotational motion allows finding the results for the angular acceleration of the system at the moment of losing power is:

              α =0.616 rad / s²

Newton's second law for rotational motion relates the torque to the moment of inertia and the angular acceleration, in the special case where the angular acceleration is zero, it is called the rotational equilibrium condition.

           [tex]\sum \tau = I \alpha[/tex]

           [tex]\tau = F \ r \ sin \theta[/tex]

where τ is the torque, F is the force, r the distance to the pivot point and θ the angle between the force, the distance, I is the moment of inertia and α is the angular acceleration.

In this case they indicate the values ​​in several different units, let's reduce to the international system of measurements (SI).

Beam mass M = 201 kg Beam length L= 59.05 ft (1m / 3.28 ft) = 18.0 m Worker and basket weight Wh = 201 lb (1N / 0.2248 lb) = 894.1 N Beam angle tea = 68.7º

We place our reference system at the base of the beam on the truck and counterclockwise turns as positive, and in the attached we see a free-body diagram of the system.

   

          [tex]W_m \ L \ sin 68.7 + Mg \ \frac{L}{2} \ sin 68.7 = I \alpha[/tex]

The moment of inertia is an additive scalar quantity, the total moment of inertia is the moment of inertia of the pen plus the moment of inertia of the man.

Moment of inertia of the beam with respect to one end.

           [tex]I_[/tex] = ⅓ ML²

moment of inertia of man

           [tex]I_m = M_m L^2[/tex]

Let's substitute

           [tex](W_m + \frac{Mg}{2} ) \ L \ sin 68.7 = (\frac{1}{3} M + m_m ) L^2 \alpha \\\alpha = \frac{(W_m + \frac{Mg}{2} ) sin 68.7 }{ (\frac{1}{3} M + m_m ) L }[/tex]

Let's calculate        

The mass of man is

           m = [tex]\frac{W_m}{g}[/tex]

           m = 894.1 / 9.8

           m = 91.2 kg

          [tex]\alpha = \frac{(894.1 +2.1 \ 9.8 ) \ sin 68.7 }{ 18.0 ( \frac{201}{3} + 91 }[/tex]

          α = 0.616 rad / s²

In conclusion using Newton's second law for rotational motion we can find the results for the angular acceleration of the system at the moment of losing power is 0.616 rad / s²

Learn more here: brainly.com/question/16428109

Helllppppp pleaseeeeee!!!

Answers

what are the directions?

A projectile is launched
horizontally from a cliff
top at 16 m/s.
Determine the values
of the velocity
components at 1-
second intervals of
time.

Answers

The vertical component of the velocity after the given time is -9.8 m/s while the horizontal component of the velocity is 16 m/s.

The given parameters;

initial horizontal velocity, vₓ = 16 m/sinitial vertical velocity, [tex]v_y =0[/tex]time interval 1 seconds

The components of the velocity can be horizontal or vertical velocity.

The vertical component of the velocity is affected by acceleration due to gravity while the horizontal component of the velocity is not affected by gravity.

The vertical component of the velocity is calculated as;

[tex]v_y = v_0_y -gt\\\\v_y = 0 - (1\times 9.8)\\\\v_y = -9.8 \ m/s[/tex]

The horizontal component of the velocity is constant since it is not affected by gravity.

The horizontal component of the velocity = 16 m/s

Thus, the vertical component of the velocity after the given time is -9.8 m/s while the horizontal component of the velocity is 16 m/s.

Learn more here:https://brainly.com/question/20349275

pleease help me please!!!​

Answers

Answer:

4 filter funnel 7 round bottom flask sorry but that is all I know

Since the bullets also have zero velocity, a more exact description of the bullet's state would be _____.

Answers

Answer:

Static equilibrium.

Explanation:

A car stops from a speed of 15 m/s in 3 s. How far did it go while stopping?

Answers

Answer:

5m/s

Explanation:

15m/s ÷ 3s = 5m/s

A train travels meters in the first second of travel, 6 meters again during the second second of travel, and 6 meters again during the third second. Its
acceleration is
O 0 m/s
6 m/s
O 12 m/s/s
18 m/s
O None of the above

Answers

Answer:

0 m/s

Explanation:

there is no acceleration, it's velocity is always 6m/s

so your answer is 0m/s

Explaining How Temperature Affects the Speed of a Wave
Does sound travel faster in a warm room or a cold room?
Explain your answer

Answers

Answer:

Warm Room

Explanation:

Sound, like any wave, is a transfer of energy from one particle to another.

As temperature rises, each particle will have more energy; they will collide into other particles more often.

As the frequency of collisions increase, the speed of which the sound wave is transferred increases.

What is one way we can use agriscience to affect photosynthesis and increase plant growth?
Removing excess water from the air
O Removing carbon dioxide from the air in greenhouses
Adding oxygen to the air in greenhouses
Adding water to soil in a precise way as needed

Answers

Answer:

casa. Completa las oraciones con el pronombre de objeto directo correcto, según el contexto.

1. Tu ropa está sucia. ¿Por qué no ____ lavas?

2. Ellos tienen muchos libros. ____ tienen en la estantería del despacho.

3. La casa de mi mamá tiene muchas ventanas, pero no ______ abre todos los días.

4. Profesora, Ud. debe usar la cochera. ¿ ______ve allí?

5. ¿Quieres ir al cine con nosotros? _____ llamamos antes de salir.

Which two limbic system structures are especially associated with forming new memories?
A. the thalamus and the hypothalamus
B. the thalamus and cerebellum
C. the hypothalamus and the substantia nigra
D. the hippocampus and amygdala

Answers

Answer:

c

Explanation:

because it was c no more explanation

Answer:

Which two limbic system structures are especially associated with forming new memories?

A. the thalamus and the hypothalamus

B. the thalamus and cerebellum

C. the hypothalamus and the substantia nigra

D. the hippocampus and amygdala

6. During an impact time casting 5 x 10-45 a gulf club exerts an average impar
force of 5000N on the ball. What linear impulse is delivered to the ball as a result?

Answers

Answer:

2.5 × 10-⁴¹ Ns

Explanation:

Impulse

I = F × t

I = 5000 N × 5 × 10-⁴⁵ s

I = 25 × 10-⁴² Ns

I = 2.5 × 10-⁴¹ Ns

#LearnWithEXO

Question
What is the total distance traveled if you drive 5.0 miles to school, and then drive 5.0 miles back home?

10.0 miles
10.0 miles

25.0 miles
25.0 miles

0 miles
0 miles

not enough information

Answers

Answer:

10 miles

Explanation:

b/c distance all what we go 5+5=10

Which of the following is an example of an acceptable time to ignore the rules and etiquette of a sport?
A.
when no one else notices
B.
when nobody really gets hurt
C.
when the outcome of the game isn't affected
D.
it's never acceptable to ignore rules and etiquette

Answers

Answer:

D-it's never acceptable to ignore rules and etiquette

Explanation:

We should know that it's never acceptable to ignore rules and etiquette.

What are the rules of etiquette in sports?

The rules of etiquette in sports are the rules of the sports that have been document by the various sports federations.

We should know that it's never acceptable to ignore rules and etiquette. These will make the the sports a lawless act and full of chaos.

Learn more about etiquette in sports:https://brainly.com/question/979191

#SPJ2

A grand prix racing car has an average lap speed of 180km/h.Each lap is 4.5 km. How many laps will it complete in 1.5 hours​

Answers

[tex]▪▪▪▪▪▪▪▪▪▪▪▪▪  {\huge\mathfrak{Answer}}▪▪▪▪▪▪▪▪▪▪▪▪▪▪[/tex]

Lap speed = 180 km/h

Each lap is about 4.5 km

So, number of laps it will complete in 1.5 hour :

[tex] \dfrac{180}{4.5} \times 1.5[/tex]

[tex] \dfrac{180}{3} [/tex]

[tex]60 \: \: laps[/tex]

Which landform is the farthest north?

Answers

The North Pole would be your answer

If a pool and a cup of water had the same temperature, which would have greater thermal energy?Why?

Answers

Despite being the same temperature, the swimming pool is a much larger thermal energy store than the beaker of water.

What is thermal energy?

Thermal energy is the energy contained within a system that controls its temperature. The flow of thermal energy is defined as heat.

Thermodynamics is a branch of physics that deals with how heat is transferred between different systems and how work is done in the process.

Thermal energy sources include natural gas, coal, and oil, as well as solar, heat pump electric, and geothermal heat.

A swimming pool and a beaker of water can both be the same temperature.

Despite being the same temperature, the swimming pool contains significantly more thermal energy than the beaker of water. A heated swimming pool's thermal store contains a lot of energy.

Thus, If a pool and a cup of water had the same temperature, then swimming pool has a much larger thermal energy.

For more details regarding thermal energy, visit:

https://brainly.com/question/11278589

#SPJ6

A plane moves 599 m northeast along a runway.
If the northern component of this displacement
is 89 m, how large is the eastern component?

Answers

The eastern component of the plane's displacement is 592.35 m.

The given parameters;

the resultant of the plane displacement = 599 mthe northern component of this displacement, Y = 89 mLet the eastern component of this displacement = X

The eastern component of the plane's displacement is calculated by applying Pythagoras theorem as follows;

X²  + Y² = 599²

X² + 89² = 599²

X² = 599²  - 89²

X² = 350880

X = √350880

X = 592.35 m

Thus, the eastern component of the plane's displacement is 592.35 m

Learn more here:https://brainly.com/question/17033407

Book A with a mass of 1.00 kg is placed on a horizontal table. Book B with a mass of 0.500 kg is placed on top of book A. What is the magnitude of the normal force exerted on book A by the table?

Answers

Answer:

Explanation:

N = mg = (1.00 + 0.500)(9.81) = 14.7 N

The magnitude of the normal force exerted on book A by the table is 14.7 N

Forces

From the question, we are to determine the magnitude of the normal force.

The normal force ca be calculated by using the formula

[tex]F_{n} = mg[/tex]

Where [tex]F_{n}[/tex] is the normal force

m is the mass

and g is the gravitational field strength ( g = 9.8 m/s²)

The mass will be addition of the masses since Book B is placed on top of the book.

Thus,

m = 1.00 kg + 0.500 kg = 1.50 kg

Then,

[tex]F_{n} = 1.5 \times 9.8[/tex]

[tex]F_{n} = 14.7 \ N[/tex]

Hence, the magnitude of the normal force exerted on book A by the table is 14.7 N

Learn more on Forces here: https://brainly.com/question/13707444

Please solve step by step

Answers

Answer:

F = 1 N

Explanation:

F = Gm₁m₂/r² + kq₁q₂/r²

F = (Gm₁m₂ + kq₁q₂) / r²

F = [6.674(10⁻¹¹)(0.50)(0.25) + 8.988(10⁹(5(10⁻⁶))(-2(10⁻⁶))] / 0.30²

F = 0.9986666667593611111...

F = 1 N

notice that the electrostatic force by itself results in an attraction of

0.99866666666666666666... N

The gravitational force is quite negligible in this example.

please can anybody tell me what are these lab equipments ​

Answers

Answer:

5 is the tripoid stand

Thanks have a bangtastic day

Answer:

hii there

5 ) tripod

6 ) test tube holder

Explanation:

Hope it helps

Have a nice day : )

Other Questions
what happens to most of the vitamin c taken as a supplement? During the Middle Ages, the city of Constantinople was-A. The largest city in the world.B. Protected by high mountains and deserts.C. The holiest city to Christians.D. The home of the popeI NEED AN ANSWER ASAP! How did the philosophers of the Enlightenment view the relationship between government and the governed? as mutually beneficial for people and government as more beneficial to the government as more beneficial to the people governed as not at all beneficial to either group please help me!! Will give Brainly if correct In lifting due to topography, what happens to air as it rises? will give brainliest to the correct answerthere are 192 candies. some are sour and some are sweet. for every 15 sour there is 1 sweet. how many candies are in the bag? how many are sweet? Please help quickly! Find the length of the third side. If necessary, write in simplest radical form. who can tell me how to be marked brainliest ion know how KWANI r u all offline "Alone" is about the peace and serenity that come from time alone. - True False Please help me figure this out, and i need to show my work A line passes through (-2,4), (-4, 8), and (n, -4). Find the value of n. What is the slope of the line represented by the equation y =4/3x-3 y = x + 3help with table and graph Any Tips for Skin Glow pls? Pls help on those questions I need help Im stressing need ASAP in 10 minutes what happens in a symbiotic relationship that is defined as mutualistic how did advancements in technology help bring a quick end to conflict in the pacific during World War II It takes 12 cups of chicken broth to make a chicken soup recipe. How much is this in quarts?Use the table below. Include the correct unit in your answer.VolumeUnit Symbol Factfluid ounce cup pint quart gallon which current supreme court justice has served the longest?